Expectation values linear harmonic oscillator

In summary: I am not joking :rolleyes::biggrin:From your answer it's not clear to me you now understand the exercise, I'm afraid. Wat is ##\phi(x)## and what is ##E## in your steady state ##\psi(x,t)## (not ##\psi(x,r)## ! ) ?
  • #1
frerk
19
1
hello :-)
here is my problem...:

1. Homework Statement


For a linear harmonic oscillator, [tex] \hat{H} = \frac{\hat{p}^2}{2m} + \frac{1}{2} m \omega^2x^2[/tex]
a) show that the expectation values for position,[tex] \bar{x},[/tex] and momentum [tex] \bar{p}[/tex] oscillate around zero with angular frequency [tex] \omega.[/tex] Hint: Use Ehrenfest Theorem

Homework Equations



Ehrenfest Theorem: [tex] \frac {d}{dt} \bar{A}= \frac{1}{i\hbar} \bar{[A,H]} = \frac{1}{i\hbar} \langle [A,H] \rangle [/tex]

The Attempt at a Solution


[/B]
So I have to make some calculations with the result [tex] \bar{x} = \bar{p} = 0.[/tex]
And I want and have to use the Ehrenfest Theorem.
With the Ehrenfest Theorem I calculated the temporal derivative of the expectation value of the position [tex] \frac {d}{dt} \bar{x}
[/tex]
and the result is zero. But that is just the derivative of the result, which I actually want to know! So.. maybe someone give me another hint? :-)
Thank you
 
Physics news on Phys.org
  • #2
frerk said:
With the Ehrenfest Theorem I calculated the temporal derivative of the expectation value of the position [tex] \frac {d}{dt} \bar{x}
[/tex]
and the result is zero.
So ##\hat{x}## commutes with ##\hat{H}##?
 
  • #3
Hi,
frerk said:
So I have to make some calculations with the result
##\bar x=\bar p=0##.​
No, you certainly don't.
For what states, exactly, are you doing this ?
 
  • #4
DrClaude said:
So ##\hat{x}## commutes with ##\hat{H}##?

oh. no. [tex] \frac{d}{dt} \bar{x} = \frac{1}{m} \bar{p} = \frac{\hbar}{im} \frac{\bar{d}}{dx} [/tex]

So I write it explicit:

[tex] \frac{d}{dt} \int_a^b \psi^*(x,t) x \psi(x,t) dx = \frac{\hbar}{im} \int_a^b dx \psi^*(x,t) \frac{d}{dx} \psi(x,t) [/tex]

And now, go on calculating this way? Is that the right way? Doesn`t feel like that...
BvU said:
Hi,
No, you certainly don't.
For what states, exactly, are you doing this ?

I thought the "problem statement" says that? that it oscillates around zero... so zero has to be the expectation value.
For which states? Do you want to know more about the "problem statement"? I have no more information. If you want to know what I think, then I would say... I shall do it for every state...?
 
Last edited:
  • #5
Check out ##{d\over dt} \bar p## too ... recognize anything ? Writing out in full is hard work, better think twice.
Remember how an arbitrary state develops ?

I ask 'for which states' because the steady states aren't oscillating in an interesting way...:rolleyes:
 
  • #6
BvU said:
Check out ##{d\over dt} \bar p## too ... recognize anything ? Writing out in full is hard work, better think twice.
Remember how an arbitrary state develops ?

I ask 'for which states' because the steady states aren't oscillating in an interesting way...:rolleyes:

I already checked it:
[tex] \frac{d}{dt} \bar{p} = -\langle \frac{dU}{dx} \rangle\quad\quad\quad U = \frac{1}{2}m\omega^2x^2[/tex]
I recognize nothing. you mean, when I compare it with: [tex] \frac{d}{dt} \bar{x} = \frac{1}{m} \bar{p} = \frac{\hbar}{im} \frac{\bar{d}}{dx} ?[/tex]

I don`t know what you want to say me with your questions...
so they are depending on time.. that means "not steady"
 
Last edited:
  • #7
So you have $$
\frac{d}{dt} \bar{x} ={1\over m} \bar{p} \quad {\text {and }} \quad \frac{d}{dt} \bar{p} = - m\omega^2 \bar{x} $$ the equations for a classical harmonic oscillator.

Steady states have ##\bar x = 0 \ ## (check this !), but a state with ##\bar x \ne 0 ## will exhibit the desired time dependence.
The exercise doesn't ask for specific states, but: can you think of such a state ?
 
  • #8
Sorry for not answering,
I had to make a break with that...

Steady states have ##\bar x = 0 \ ## (check this !)

ok. here is my steady state: [tex] \psi(x,r) = \phi(x)e^{-iEt/\hbar} [/tex]

[tex] \bar{x} = \int_{-\infty}^{+\infty} dx \phi^*(x)e^{iEt/\hbar} x \phi(x)e^{-iEt/\hbar} = = \int_{-\infty}^{+\infty} |\phi(x)|^2 x [/tex]
And this has to be zero. Because an integral is just the area between the grap of a function and the x-axis. and the "x" brings a "point reflection.

but a state with ##\bar x \ne 0 ## will exhibit the desired time dependence.

okay. So the result can be derived temporal and that would be the same than [tex] 1/m * \bar{p} [/tex]

The exercise doesn't ask for specific states, but: can you think of such a state ?

no specific one... but a state which has not the form [tex] \psi(x,r) = \phi(x)e^{-iEt/\hbar} [/tex] or more general: a state, which propability density [tex] | \psi(x)|^2 [/tex] doesn`t depend from time.

for example...: [tex] \psi(x,r) = \phi(x)(Et/\hbar) [/tex]

sorry, but I am not joking :rolleyes::biggrin:
 
  • #9
From your answer it's not clear to me you now understand the exercise, I'm afraid. Wat is ##\phi(x)## and what is ##E## in your steady state ##\psi(x,t)## (not ##\psi(x,r)## ! ) ?

And what do you mean with ##\psi(x,r) = \phi(x)(Et/\hbar)## ? It's not a solution to the time-dependent SE !
 
  • Like
Likes frerk
  • #10
BvU said:
From your answer it's not clear to me you now understand the exercise, I'm afraid. Wat is ##\phi(x)## and what is ##E## in your steady state ##\psi(x,t)## (not ##\psi(x,r)## ! ) ?

And what do you mean with ##\psi(x,r) = \phi(x)(Et/\hbar)## ? It's not a solution to the time-dependent SE !

In my first part, I just checked, whether [tex] \bar{x} = 0 [/tex] for steady states.

And yes, [tex] \psi(x,t) = \phi(x)(Et/\hbar) [/tex] is not a solution of time-dependent SE.
And also yes, I wanted to write [tex] \psi(x,t) [/tex] and not [tex] \psi(x,r) [/tex], but the second time I wrote it I just copied it, so the mistake happened twice.

I try what to explain, what to do in this exercise and what we have reached so far:smile:

We have to show, that the expectation value for position and for momentum oscillate around zero with angular frequency omega.

We have the first half of our solution: The solution for steady states: The expectation values for position [tex] \bar{x} [/tex] is zero, (I showed that graphical) and because of [tex] \frac{d}{dt} \bar{x} = \frac{1}{m} \bar{p} ,[/tex] also the expectation value of the momentum is zero.

Now we have to do the second half: Show the expectation values for not-steady states?

The exercise doesn't ask for specific states, but: can you think of such a state ?

My last guess was bad. I have no second guess...
 
  • #11
Steady states for the harmonic oscillator are eigenfunctions of the Hamiltonian: ##H\phi(x) = E\phi(x)##. The solutions of the time-independen Schroedinger equation are distinct wavefunctions ##\phi_n(r)## with distinct energies ##E_n##. Their expectation value for ##\bar x## is zero because ##\phi_n^* \phi_n ## is symmetric and x is antisymmetric.

What about e.g. ##\phi_1^* \phi_0 ## ?

Arbitrary general states ##\Phi(r)## can be written as ##\sum_n C_n\phi_n## with ##C_n## derived from ##\Phi(r)## at t= 0 and they develop in time as
##\Phi(r,t) = \sum_n C_n\phi_n e^{-{i\over \hbar} E_n t}## .

A simple case is ##\Phi(r) = {1\over \sqrt 2} \left ( \phi_0 + \phi_1 \right )## where the time development of ##\bar x## no longer vanishes. Doing the integrals with the Hermite polynomials is hard work (but worth it). Easier to use ladder operators, but you may not have seen those yet.

Check the first picture in wikipedia
 
  • #12
BvU said:
Steady states for the harmonic oscillator are eigenfunctions of the Hamiltonian: ##H\phi(x) = E\phi(x)##. The solutions of the time-independen Schroedinger equation are distinct wavefunctions ##\phi_n(r)## with distinct energies ##E_n##. Their expectation value for ##\bar x## is zero because ##\phi_n^* \phi_n ## is symmetric and x is antisymmetric.

That is the same, I wanted to say with that, just graphical:

BvU said:
ok. here is my steady state: [tex] \psi(x,t) = \phi(x)e^{-iEt/\hbar} [/tex]

[tex] \bar{x} = \int_{-\infty}^{+\infty} dx \phi^*(x)e^{iEt/\hbar} x \phi(x)e^{-iEt/\hbar} = \int_{-\infty}^{+\infty} |\phi(x)|^2 x [/tex]
And this has to be zero. Because an integral is just the area between the graph of a function and the x-axis. and the "x" brings a "point reflection.

I make an example. I calculate the expectation value for the position for ## \psi_1 ## which is:

[tex] \psi_1(\zeta) = (\frac{m\omega}{\pi\hbar})^{1/4} \frac{1}{\sqrt{2^11!}} H_1(\zeta) e^{-0,5\zeta^2}
= (\frac{m\omega}{\pi\hbar})^{1/4} \frac{1}{\sqrt{2}} 2\zeta e^{-0,5\zeta^2} = (\frac{m\omega}{\pi\hbar})^{1/4} \sqrt{2} \zeta e^{-0,5\zeta^2} [/tex]
(Is that right? because my skript says sth. different and I don`t find a proper source elsewere)
[tex] \rightarrow \bar{\zeta} = \int_{-\infty}^{+\infty} d\zeta (\frac{m\omega}{\pi\hbar})^{1/2} 2 \zeta^2 e^{-\zeta^2} \zeta = 0, [/tex] because of the two made explanations. (the one part of this integral is symmetric, the other one antisymmetric...) and because of [tex] \zeta = \sqrt{\frac{m\omega}{\hbar}} x [/tex] also the expectation value of x is zero.

BvU said:
What about e.g. ##\phi_1^* \phi_0 ## ?

[tex] \rightarrow \bar{\zeta} = \int_{-\infty}^{+\infty} (\frac{m\omega}{\pi\hbar})^{1/4} \sqrt{2} \zeta e^{-0,5\zeta^2} \cdot \zeta \cdot (\frac{m\omega}{\pi\hbar})^{1/4} e^{-0,5\zeta^2} = \int_{-\infty}^{+\infty} (\frac{m\omega}{\pi\hbar})^{1/2} e^{-\zeta^2} \sqrt{2} \zeta^2 = (\frac{m\omega}{\pi\hbar})^{1/2} \frac{1}{\sqrt{2}} \sqrt{\pi} [/tex] The integral was solved with a gauss-integral... so not zero..
is there a rule for that?

just the integral ##\int \phi_1^* \phi_0 ## would be zero. The integral for Hermite polynomials says that.

BvU said:
Arbitrary general states ##\Phi(r)## can be written as ##\sum_n C_n\phi_n## with ##C_n## derived from ##\Phi(r)## at t= 0 and they develop in time as
##\Phi(r,t) = \sum_n C_n\phi_n e^{-{i\over \hbar} E_n t}## .

A simple case is ##\Phi(r) = {1\over \sqrt 2} \left ( \phi_0 + \phi_1 \right )## where the time development of ##\bar x## no longer vanishes. Doing the integrals with the Hermite polynomials is hard work (but worth it). Easier to use ladder operators, but you may not have seen those yet.

Check the first picture in wikipedia

so the time development vanishes for ##\phi_n^* \phi_n ## and ##\phi_n^* \phi_m ## because they are steady states. ##\Phi(r) = {1\over \sqrt 2} \left ( \phi_0 + \phi_1 \right )## is also solution of schroedinger equation, but the time development doesn`t vanish.

so you mean I shall calculate this:::[tex] \bar{\zeta} = \int_{-\infty}^{+\infty} \frac{1}{\sqrt{2} } (\frac{m\omega}{\pi\hbar})^{1/4} e^{-0,5\zeta^2} e^{iE_0t/\hbar} + (\frac{m\omega}{\pi\hbar})^{1/4} \sqrt{2} \zeta e^{-0,5\zeta^2} e^{iE_1t/\hbar} \cdot \zeta \cdot \frac{1}{sqrt{2}} (\frac{m\omega}{\pi\hbar})^{1/4} e^{-0,5\zeta^2} e^{-iE_0t/\hbar} + (\frac{m\omega}{\pi\hbar})^{1/4} \sqrt{2} \zeta e^{-0,5\zeta^2} e^{-iE_1t/\hbar} d\zeta [/tex]?

So the answer to the question, how the expectation values for a linear harmonic oscillator are, is:

## \bar{x} ## is zero for ##\phi_n^* \phi_n ## and a constant for ##\phi_n^* \phi_m ##.
## \bar{p} ## is zero for ##\phi_n^* \phi_n ## and for ##\phi_n^* \phi_m ##
and ## \bar{x} ## + ## \bar{p} ## aren`t zero, when we use a combination of different states?
 
  • #13
Oh boy, what a big post. You certainly did a lot of ##\TeX##ing there !

Good work, but some things need to be revisited:

Most importantly: we overlooked the time-dependent part when calculating ##\bar x##. No disaster for $$
\left (\phi_0(x) e^{-{i\over \hbar}E_0t} \right )^* \;x\; \left (\phi_0(x) e^{-{i\over \hbar}E_0t} \right ) $$ but you can see a ##e^{-{i\over \hbar}(E_1-E_0) t} ## appearing for ##\phi_1^* \phi_0##

And the only interesting terms in ##\bar x (t) ## for ##
\Phi(r) = {1\over \sqrt 2} \left ( \phi_0 + \phi_1 \right )## are ##\phi_1^* \phi_0## and ##\phi_0^* \phi_1## which are equal, so you have to calculate only one.

And Hermite polynomials have some useful properties that allow partial integration. But you can also be satisfied with an outcome like constant ##* \cos(\omega t)##
 
  • Like
Likes frerk
  • #14
BvU said:
Oh boy, what a big post. You certainly did a lot of ##\TeX##ing there !

Good work, but some things need to be revisited:

Thank you for your recognition :redface:

BvU said:
Most importantly: we overlooked the time-dependent part when calculating ##\bar x##. No disaster for $$
\left (\phi_0(x) e^{-{i\over \hbar}E_0t} \right )^* \;x\; \left (\phi_0(x) e^{-{i\over \hbar}E_0t} \right ) $$ but you can see a ##e^{-{i\over \hbar}(E_1-E_0) t} ## appearing for ##\phi_1^* \phi_0##
I see that. Just the "minus" is wrong I think.
[tex] \rightarrow \bar{\zeta} = \int_{-\infty}^{+\infty} (\frac{m\omega}{\pi\hbar})^{1/4} \sqrt{2} \zeta e^{-0,5\zeta^2} e^{+iE_1t/\hbar}
\cdot \zeta \cdot
(\frac{m\omega}{\pi\hbar})^{1/4} e^{-0,5\zeta^2} e^{-iE_0t/\hbar}
= \int_{-\infty}^{+\infty} (\frac{m\omega}{\pi\hbar})^{1/2} e^{-\zeta^2} \sqrt{2} \zeta^2 e^{i(E_1-E_0)t/\hbar} = (\frac{m\omega}{\pi\hbar})^{1/2} \frac{1}{\sqrt{2}} \sqrt{\pi} e^{i(E_1-E_0)t/\hbar} [/tex]

[tex] \rightarrow \bar{x} = \frac{1}{\sqrt{2}} e^{i(E_1-E_0)t/\hbar} [/tex]
[tex] \rightarrow \bar{p} = m \frac{i(E_1-E_0)}{\hbar} \frac{1}{\sqrt{2}} e^{i(E_1-E_0)t/\hbar} [/tex]

So this are the both expectation values...

How about more general: ## \int_{-\infty}^{+\infty} \phi_n^* \cdot \zeta \cdot \phi_m ##

would it be: ## \bar{x} = \frac{1}{\sqrt{2}} e^{i(E_n-E_m)t/\hbar} ## ?

My problem is now...
I understand the calculations and what want to show for ## \phi_n^*\phi_n ##. And how their graphs look like.

But what is with ## \phi_n^*\phi_m ##? The normalization "says", that it is zero. But what does that say? Is ##\int \phi_n^*\phi_m ## the probability that both states exist at the same time? Then zero would make sense. But what does the expectation value for ## \int \phi_n^*\phi_m ## say then say?

BvU said:
And the only interesting terms in ##\bar x (t) ## for ##
\Phi(r) = {1\over \sqrt 2} \left ( \phi_0 + \phi_1 \right )## are ##\phi_1^* \phi_0## and ##\phi_0^* \phi_1## which are equal, so you have to calculate only one.

fortunately that makes sense for me. So:

[tex] \bar{\zeta} = 2 e^{i(E_= - E_1) t / \hbar} \int \phi_0 \phi_1 \zeta d\zeta = 2 e^{i(E_0 - E_1) t / \hbar} (\frac{m\omega}{\pi\hbar})^{1/2} \frac{1}{\sqrt{2}} \sqrt{\pi} [/tex]

BvU said:
And Hermite polynomials have some useful properties that allow partial integration. But you can also be satisfied with an outcome like constant ##* \cos(\omega t)##

I already calculated it for a previous Thread. And I used a Gauss-integral. okay..

[tex] \rightarrow \bar{x} = \sqrt{2} e^{i(E_0 - E_1) t / \hbar} [/tex]

So. This is a next result. But there are a lot of more.. for example a combination of 3 different terms..
 
  • #15
frerk said:
Just the "minus" is wrong I think
Probably. But your ##
\rightarrow \bar{x} = \frac{1}{\sqrt{2}} e^{i(E_1-E_0)t/\hbar}## doesn't ring too good either: there are two imaginary terms (one from ##\ \phi_1^*x\phi_0\ ## and one from ##\ \phi_0^* x \phi_1\ ##and together they should yield a real expectation value ##\bar{x} = \bar x_0 \cos \omega t ##.

[edit] can't be all; there should also be some ## \bar p_0/m \sin \omega t ##.
frerk said:
How about more general: ## \int_{-\infty}^{+\infty} \phi_n^* \cdot \zeta \cdot \phi_m##
From the back of my mind (rusty and dusty there) I come up with the expectation :smile: that they all give 0 except when ##|n-m| = 1##. Will do some looking up later ...
 
Last edited:
  • Like
Likes frerk
  • #16
I googled around but can't find a treatment based on the explicit wave functions/Hermite polynomials: everybody (http://physics.ucdavis.edu/classes/physics115a/harmonic.pdf, ucsd, usu, mit, Reed) uses ladder operators (for good reasons: much less tedious and very useful for later). Check out one or two of them to see if we didn't make too big mistakes in this thread :smile:

In view of the fact that your exercise was effectively done already with post #6 I propose we call it a day :smile:

Just one loose end to clear up:
frerk said:
But what is with ## \phi_n^*\phi_m## ? The normalization "says", that it is zero. But what does that say? Is it the probability that both states exist at the same time? Then zero would make sense. But what does the expectation value for ## \int \phi_n^*\phi_m ## then say?
It says that eigenfunctions with different eigenvalues are orthogonal. The probablility to find eigenvalue n when the system is in the eigenstate m is zero when n and m are different. This is in line with the time-independency of eigenstates of the Hamiltonian. Important concept, so make sure you have no doubts or lingering questions left.

[edit] looks like I skipped something: H needs to be unitary for orthogonality.

But since the Schroedinger equation is linear, solutions (i.e. possible states) can be linear combinations of eigenstates. And such solutions are not eigenfunctions of the Hamiltonian, so they do develop in time (as we've seen in this exercise). And this time development is unique (fortunately) thanks to the orthogonalty of the eigenstates.

I liked this exercise/thread a great deal (as a refresher also :rolleyes:).
 
Last edited:
  • Like
Likes frerk
  • #17
BvU said:
I googled around but can't find a treatment based on the explicit wave functions/Hermite polynomials: everybody (http://physics.ucdavis.edu/classes/physics115a/harmonic.pdf, ucsd, usu, mit, Reed) uses ladder operators (for good reasons: much less tedious and very useful for later). Check out one or two of them to see if we didn't make too big mistakes in this thread :smile:

Yes, I have also googled a lot, and they almost use the ladder operators...

BvU said:
In view of the fact that your exercise was effectively done already with post #6 I propose we call it a day :smile:

Sure :smile:

BvU said:
It says that eigenfunctions with different eigenvalues are orthogonal. The probablility to find eigenvalue n when the system is in the eigenstate m is zero when n and m are different. This is in line with the time-independency of eigenstates of the Hamiltonian. Important concept, so make sure you have no doubts or lingering questions left.

First half makes very much sense for me, and I have to read more about the second half :smile:
BvU said:
I liked this exercise/thread a great deal (as a refresher also :rolleyes:).

Yes, for me too. You helped me a lot :smile:
greetings from germany :smile:
 

Related to Expectation values linear harmonic oscillator

What is the linear harmonic oscillator model?

The linear harmonic oscillator model is a mathematical model used in physics to describe the motion of a particle, such as an electron, in a potential energy field. It assumes that the restoring force acting on the particle is directly proportional to its displacement from equilibrium.

What is the significance of expectation values in the linear harmonic oscillator model?

In the linear harmonic oscillator model, expectation values are used to predict the average value of a physical quantity, such as position or momentum, over many measurements. They are important because they allow us to make predictions about the behavior of a system without having to measure every single instance.

How do you calculate expectation values in the linear harmonic oscillator model?

To calculate expectation values in the linear harmonic oscillator model, you first need to know the wave function of the system. Then, you can use the operator method or the integral method to find the expectation value of a given physical quantity. The integral method involves integrating the product of the wave function and the operator, while the operator method involves applying the operator to the wave function.

What is the relationship between the expectation values and the uncertainty principle in the linear harmonic oscillator model?

In the linear harmonic oscillator model, the uncertainty principle states that the product of the uncertainty in position and momentum is equal to or greater than a certain value. The expectation values for position and momentum can be used to calculate this minimum value, known as the uncertainty limit, and to determine the range of possible values for these quantities.

How does the linear harmonic oscillator model relate to real-world systems?

The linear harmonic oscillator model is a simplified version of real-world systems, but it is still applicable in many cases. For example, it can be used to model the vibrations of molecules, the oscillations of a pendulum, or the behavior of a spring. It is also a key component in more complex models, such as the quantum harmonic oscillator, which is used to describe the behavior of atoms and subatomic particles.

Similar threads

  • Advanced Physics Homework Help
Replies
1
Views
787
  • Advanced Physics Homework Help
Replies
16
Views
795
  • Advanced Physics Homework Help
Replies
31
Views
3K
  • Advanced Physics Homework Help
Replies
7
Views
896
  • Advanced Physics Homework Help
Replies
1
Views
2K
  • Advanced Physics Homework Help
Replies
1
Views
2K
Replies
1
Views
1K
Replies
11
Views
1K
  • Advanced Physics Homework Help
Replies
2
Views
478
Back
Top